8.9: The Allowed Values of J - the Total Angular Momentum Quantum Number (2024)

  1. Last updated
  2. Save as PDF
  • Page ID
    210847
  • \( \newcommand{\vecs}[1]{\overset { \scriptstyle \rightharpoonup} {\mathbf{#1}}}\)

    \( \newcommand{\vecd}[1]{\overset{-\!-\!\rightharpoonup}{\vphantom{a}\smash{#1}}} \)

    \( \newcommand{\id}{\mathrm{id}}\) \( \newcommand{\Span}{\mathrm{span}}\)

    ( \newcommand{\kernel}{\mathrm{null}\,}\) \( \newcommand{\range}{\mathrm{range}\,}\)

    \( \newcommand{\RealPart}{\mathrm{Re}}\) \( \newcommand{\ImaginaryPart}{\mathrm{Im}}\)

    \( \newcommand{\Argument}{\mathrm{Arg}}\) \( \newcommand{\norm}[1]{\| #1 \|}\)

    \( \newcommand{\inner}[2]{\langle #1, #2 \rangle}\)

    \( \newcommand{\Span}{\mathrm{span}}\)

    \( \newcommand{\id}{\mathrm{id}}\)

    \( \newcommand{\Span}{\mathrm{span}}\)

    \( \newcommand{\kernel}{\mathrm{null}\,}\)

    \( \newcommand{\range}{\mathrm{range}\,}\)

    \( \newcommand{\RealPart}{\mathrm{Re}}\)

    \( \newcommand{\ImaginaryPart}{\mathrm{Im}}\)

    \( \newcommand{\Argument}{\mathrm{Arg}}\)

    \( \newcommand{\norm}[1]{\| #1 \|}\)

    \( \newcommand{\inner}[2]{\langle #1, #2 \rangle}\)

    \( \newcommand{\Span}{\mathrm{span}}\) \( \newcommand{\AA}{\unicode[.8,0]{x212B}}\)

    \( \newcommand{\vectorA}[1]{\vec{#1}} % arrow\)

    \( \newcommand{\vectorAt}[1]{\vec{\text{#1}}} % arrow\)

    \( \newcommand{\vectorB}[1]{\overset { \scriptstyle \rightharpoonup} {\mathbf{#1}}}\)

    \( \newcommand{\vectorC}[1]{\textbf{#1}}\)

    \( \newcommand{\vectorD}[1]{\overrightarrow{#1}}\)

    \( \newcommand{\vectorDt}[1]{\overrightarrow{\text{#1}}}\)

    \( \newcommand{\vectE}[1]{\overset{-\!-\!\rightharpoonup}{\vphantom{a}\smash{\mathbf {#1}}}} \)

    \( \newcommand{\vecs}[1]{\overset { \scriptstyle \rightharpoonup} {\mathbf{#1}}}\)

    \( \newcommand{\vecd}[1]{\overset{-\!-\!\rightharpoonup}{\vphantom{a}\smash{#1}}} \)

    We need to be able to identify the electronic states that result from a given electron configuration and determine their relative energies. An electronic state of an atom is characterized by a specific energy, wavefunction (including spin), electron configuration, total angular momentum, and the way the orbital and spin angular momenta of the different electrons are coupled together. There are two descriptions for the coupling of angular momentum. One is called j-j coupling, and the other is called L-S coupling. The j-j coupling scheme is used for heavy elements (z > 40) and the L-S coupling scheme is used for the lighter elements. Only L-S coupling is discussed below.

    L-S Coupling of Angular Momenta

    L-S coupling also is called R-S or Russell-Saunders coupling. In L-S coupling, the orbital and spin angular momenta of all the electrons are combined separately

    \[L = \sum _i l_i \label{8.11.3}\]

    \[S = \sum _i s_i \label{8.11.4}\]

    The total angular momentum vector then is the sum of the total orbital angular momentum vector and the total spin angular momentum vector.

    \[J = L + S \label{8.11.5}\]

    The total angular momentum quantum number parameterizes the total angular momentum of a given particle, by combining its orbital angular momentum and its intrinsic angular momentum (i.e., its spin). Due to the spin-orbit interaction in the atom, the orbital angular momentum no longer commutes with the Hamiltonian, nor does the spin.

    8.9: The Allowed Values of J - the Total Angular Momentum Quantum Number (1)

    However the total angular momentum \(J\) does commute with the Hamiltonian and so is a constant of motion (does not change in time). The relevant definitions of the angular momenta are:

    Orbital Angular Momentum

    \[|\vec{L}| = \hbar \sqrt{\ell(\ell+1)}\]

    with its projection on the z-axis \[L_z = m_\ell \hbar\]

    Spin Angular Momentum

    \[ |\vec{S}| = \hbar \sqrt{s(s+1)}\]

    with its projection on the z-axis \[ S_z = m_s \hbar \]

    Total Angular Momentum

    \[ |\vec{J}| = \hbar \sqrt{j(j+1)}\]

    with its projection on the z-axis \[ J_z = m_j \hbar \]

    where

    • \(l\) is the azimuthal quantum number of a single electron,
    • \(s\) is the spin quantum number intrinsic to the electron,
    • \(j\) is the total angular momentum quantum number of the electron,

    The quantum numbers take the values:

    \[\begin{align} & m_\ell \in \{ -\ell, -(\ell-1) \cdots \ell-1, \ell \} , \quad \ell \in \{ 0,1 \cdots n-1 \} \\& m_s \in \{ -s, -(s-1) \cdots s-1, s \} , \\& m_j \in \{ -j, -(j-1) \cdots j-1, j \} , \\& m_j=m_\ell+m_s, \quad j=|\ell+s|\\\end{align} \]

    and the magnitudes are:

    \[\begin{align} & |\textbf{J}| = \hbar\sqrt{j(j+1)} \\& |\textbf{J}_1| = \hbar\sqrt{j_1(j_1+1)} \\& |\textbf{J}_2| = \hbar\sqrt{j_2(j_2+1)} \\\end{align} \]

    in which

    \[ j \in \{ |j_1 - j_2|, |j_1 - j_2| - 1 \cdots j_1 + j_2 - 1, j_1 + j_2 \} \,\! \]

    This process may be repeated for a third electron, then the fourth etc. until the total angular momentum has been found.

    8.9: The Allowed Values of J - the Total Angular Momentum Quantum Number (2)

    The result of these vector sums is specified in a code that is called a Russell-Saunders term symbol, and each term symbol identifies an energy level of the atom. Consequently, the energy levels also are called terms. A term symbol has the form \(^{2s+1} L_J\) where the code letter that is used for the total orbital angular momentum quantum number L = 0, 1, 2, 3, 4, 5 is S, P, D, F, G, H, respectively. Note how this code matches that used for the atomic orbitals. The superscript \(2S+1\) gives the spin multiplicity of the state, where S is the total spin angular momentum quantum number. The spin multiplicity is the number of spin states associated with a given electronic state. In order not to confuse the code letter S for the orbital angular momentum with the spin quantum number S, you must examine the context in which it is used carefully. In the term symbol, the subscript J gives the total angular momentum quantum number. Because of spin-orbit coupling, only \(J\) and \(M_j\) are valid quantum numbers, but because the spin-orbit coupling is weak \(L\), \(M_l\), \(S\), and \(m_s\) still serve to identify and characterize the states for the lighter elements.

    For example, the ground state, i.e. the lowest energy state, of the hydrogen atom corresponds to the electron configuration in which the electron occupies the 1s spatial orbital and can have either spin \(\alpha\) or spin \(\beta\). The term symbol for the ground state is \(^2 S_{1/2}\), which is read as “doublet S 1/2”. The spin quantum number is 1/2 so the superscript 2S+1 = 2, which gives the spin multiplicity of the state, i.e. the number of spin states equals 2 corresponding to \(\alpha\) and \(\beta\). The S in the term symbol indicates that the total orbital angular momentum quantum number is 0 (For the ground state of hydrogen, there is only one electron and it is in an s-orbital with \(l = 0\) ). The subscript ½ refers to the total angular momentum quantum number. The total angular momentum is the sum of the spin and orbital angular momenta for the electrons in an atom. In this case, the total angular momentum quantum number is just the spin angular momentum quantum number, ½, since the orbital angular momentum is zero. The ground state has a degeneracy of two because the total angular momentum can have a z-axis projection of \(+\frac {1}{2} \hbar\) or \(-\frac {1}{2} \hbar\), corresponding to \(m_J\) = +1/2 or -1/2 resulting from the two electron spin states \(\alpha\) and \(\beta\). We also can say, equivalently, that the ground state term or energy level is two-fold degenerate.

    Exercise \(\PageIndex{1}\)

    Write the term symbol for a state that has 0 for both the spin and orbital angular momentum quantum numbers.

    Exercise \(\PageIndex{2}\)

    Write the term symbol for a state that has 0 for the spin and 1 for the orbital angular momentum quantum numbers

    Contributors

    8.9: The Allowed Values of J - the Total Angular Momentum Quantum Number (2024)

    FAQs

    What are the allowed values of total angular momentum quantum number? ›

    The possible values for the orbital angular momentum quantum number are l = 1 and l = 2. (j = l + s, ..., l - s; j = l + ½, l - ½, implies l = 1 or l = 2.) The parity of the orbital state is (-1)l. If the parity is odd, we have l = 1, if the parity is even, we have l = 2.

    How to calculate total angular momentum quantum number? ›

    The total angular momentum is the sum of the spin and orbital angular momenta for the electrons in an atom. In this case, the total angular momentum quantum number is just the spin angular momentum quantum number, ½, since the orbital angular momentum is zero.

    What is the possible value of the angular momentum quantum number? ›

    The angular momentum quantum number, signified by l, describes the general shape or region an electron occupies—its orbital shape. The value of l depends on the value of the principal quantum number, n. The angular momentum quantum number can have positive values of zero to (n−1).

    What is total angular momentum j? ›

    The total angular momentum has the magnitude Square root of√J(J + 1) (ℏ), in which J can take any positive value from L + S to |L − S| in integer steps; i.e., if L = 1 and S = 3/2, J can be 5/2, 3/2, or 1/2.

    Which of the following are allowed values for the angular momentum quantum number? ›

    Intrinsic Spin
    NameSymbolAllowed Values
    Principal quantum numbern1, 2, 3, . . .
    Angular momentuml0, 1, 2, . . .n−1
    Angular momentum projectionml−l,−l+1, . . .,−1, 0, 1, . . .,l−1,l(or0, ±1, ±2, . . .,±l)
    Spin 1s1/2(electrons)
    1 more row

    What are the allowed values of S? ›

    The allowed values for s are non-negative integers or half-integers. Fermions have half-integer values, including the electron, proton and neutron which all have s = ++ 1 /2 . Bosons such as the photon and all mesons) have integer spin values.

    How do you find the value of angular momentum? ›

    Angular momentum is given by L=Iω, thus, substituting the values we get L=0.04 kg.m².s-¹.

    What is the formula for the total angular momentum of a system? ›

    If we have a system of N particles, each with position vector from the origin given by →ri and each having momentum →pi, then the total angular momentum of the system of particles about the origin is the vector sum of the individual angular momenta about the origin. That is, →L=→l1+→l2+⋯+→lN.

    What is the minimum value of the angular momentum quantum number? ›

    The correct Answer is:hπ

    Step by step video, text & image solution for The minimum angular momentum of an electron with the magnetic quantum numbers -1,0,+1 by Chemistry experts to help you in doubts & scoring excellent marks in Class 10 exams.

    What is the rule for the values allowed for the angular momentum quantum number L )? ›

    Rules Governing the Allowed Combinations of Quantum Numbers

    The angular quantum number (l) can be any integer between 0 and n - 1. If n = 3, for example, l can be either 0, 1, or 2. The magnetic quantum number (m) can be any integer between -l and +l.

    What is the maximum angular momentum quantum number? ›

    The angular momentum quantum number, l, (also referred to as the secondary quantum number or azimuthal quantum number) describes the shape of the orbital that an electron occupies. The lowest possible value of l is 0, and its highest possible value, depending on the principal quantum number, is n - 1.

    What is the formula for angular momentum quantum? ›

    The magnitude of angular momentum is given by L=√l(l+1)h2π(l=0,1,2,...,n−1), where l is the angular momentum quantum number.

    What is the angular momentum in terms of J? ›

    Finally, there is total angular momentum J, which combines both the spin and orbital angular momentum of all particles and fields. (For one particle, J = L + S.)

    How do I find angular momentum? ›

    Angular momentum is the angular equivalent of linear momentum (p=mv) and is given by L=Iω, where I(kgm2) is the moment of inertia of an object about an axis and ω(rads−1) is its angular velocity about that axis. Angular momentum has the units kgm2s−1.

    What are the restrictions on the angular momentum quantum number? ›

    Once nn is known, the values of the angular momentum quantum number are limited to l=1, 2, 3, …,n−1l=1, 2, 3, …,n−1. For a given value of ll, the angular momentum projection quantum number can have only the values ml=−l,−l+1, …,−1, 0, 1, …,l−1,lml=−l,−l+1, …,−1, 0, 1, …,l−1,l.

    What are the allowed values of the angular momentum quantum number l if the principal quantum number is n 4? ›

    Answer and Explanation:

    For the angular momentum quantum number, ℓ, the possible values range from 0 to n - 1. As such, the possible values of ℓ when n = 4 are 0, 1, 2, and 3.

    Can angular momentum quantum number be greater than 3? ›

    Once n is known, the values of the angular momentum quantum number are limited to l=1,2,3,...,n−1. For a given value of l, the angular momentum projection quantum number can have only the values ml=−l,−l+1,...,−1,0,1,...,l−1,l. Electron spin is independent of n, l, and ml, always having s=1/2.

    Top Articles
    Latest Posts
    Article information

    Author: Tyson Zemlak

    Last Updated:

    Views: 6431

    Rating: 4.2 / 5 (43 voted)

    Reviews: 90% of readers found this page helpful

    Author information

    Name: Tyson Zemlak

    Birthday: 1992-03-17

    Address: Apt. 662 96191 Quigley Dam, Kubview, MA 42013

    Phone: +441678032891

    Job: Community-Services Orchestrator

    Hobby: Coffee roasting, Calligraphy, Metalworking, Fashion, Vehicle restoration, Shopping, Photography

    Introduction: My name is Tyson Zemlak, I am a excited, light, sparkling, super, open, fair, magnificent person who loves writing and wants to share my knowledge and understanding with you.